LSAT and Law School Admissions Forum

Get expert LSAT preparation and law school admissions advice from PowerScore Test Preparation.

 Administrator
PowerScore Staff
  • PowerScore Staff
  • Posts: 8917
  • Joined: Feb 02, 2011
|
#26037
Complete Question Explanation
(The complete setup for this game can be found here: lsat/viewtopic.php?t=10807)

The correct answer choice is (A)

If aisle 3 contains P only, then M must be located in aisle 2, and F—in aisle 1 in accord with the second rule:
oct12_game_3_#14_diagram_1.png
This setup reveals that answer choice (A) must be true, and is therefore correct.

Answer choices (B) and (E) could be true, but because H and S can both be located in either aisle 1 or 2, neither of them must be true. Answer choice (C) cannot be true. Answer choice (D) could be true, but need not be true because R could also be in aisle 2.
You do not have the required permissions to view the files attached to this post.

Get the most out of your LSAT Prep Plus subscription.

Analyze and track your performance with our Testing and Analytics Package.